Rosh IM Boost Exam

अब Quizwiz के साथ अपने होमवर्क और परीक्षाओं को एस करें!

A 58-year-old woman presents to the hospital with dyspnea that started earlier today. She also reports right-sided chest pain that is worse with inspiration. The patient takes hormone replacement therapy for menopausal symptoms and reports no prior abnormal episodes of bleeding. The patient has a medical history of hypertension treated with amlodipine. Vital signs include a BP of 130/90 mm Hg, HR of 115 bpm, RR of 24/minute, T of 98.6°F, and SpO2 of 94% on room air. Physical examination reveals mild tachypnea with lungs clear to auscultation. The patient has a regular tachycardic rate. The patient's right lower extremity has asymmetric edema and right calf tenderness. The patient has calf pain on the right side with passive dorsiflexion of the foot. The patient's ECG is shown above. Chest X-ray reveals no pneumothorax, pulmonary edema, or infiltration. The patient's initial troponin I is 0.1 ng/mL. Further laboratory results induce a platelet count of 300,000/μL and creatinine of 0.8 mg/dL. D-dimer and imaging results are pending. Which of the following is the recommended treatment if the suspected diagnosis is confirmed? A. Anticoagulation therapy with enoxaparin B. Anticoagulation th

A. .Anticoagulation therapy with enoxaparin

A 62-year-old woman presents to the clinic with intermittent episodes of facial flushing for the past 3 weeks. She also reports diarrhea over the same time period. She describes the episodes of flushing as having sudden onset and lasting 30 seconds. She has a medical history of hypertension, for which she takes lisinopril. Vital signs include a BP of 110/70 mm Hg, HR of 99 bpm, RR of 20/minute, T of 98.6°F, and SpO2 of 98% on room air. During the physical examination, the patient has an episode of facial flushing, and her face appears violaceous in color. She has expiratory wheezing during the episode. Which of the following is the initial diagnostic test for the suspected condition? A. 24-hour urinary excretion of 5-hydroxyindoleacetic acid B. 24-hour urine fractionated metanephrines and catecholamines C. Chromogranin concentration D. Serum gastrin levels

A. 24-hour urinary excretion of 5-hydroxyindoleacetic acid

A 33-year-old man who is otherwise healthy presents to the office with complaints of easy fatigability and shortness of breath with exercise that has worsened over the past six months. He is active and has been mountain biking for years but recently is unable to bike for more than five minutes without stopping to rest. An echocardiogram reveals a bicuspid aortic valve. Which of the following would likely be noted on auscultation? A. A harsh crescendo-decrescendo systolic murmur over the right second intercostal space radiating to the carotid arteries B. A holosystolic murmur heard over the apex of the heart radiating to the back C. A loud first heart sound with an opening snap and diastolic rumble D. A midsystolic click followed by a high-pitched late systolic murmur at the cardiac apex

A. A harsh crescendo-decrescendo systolic murmur over the right second intercostal space radiating to the carotid arteries

A 68-year-old man presents to the emergency department with fever and productive cough. The initial vital signs show the patient has a low-grade fever, respiratory rate of 30/min, and normal heart rate and blood pressure. He is alert and oriented and otherwise healthy. Laboratory findings are unremarkable. Chest X-ray shows an infiltrate in the right lower lobe. What is the best treatment plan based on the above information? A. Admission to the floor and start empiric antibiotics B. Admission to the floor and wait for culture results to guide antibiotics C. Admission to the intensive care unit and intravenous antibiotics D. Discharge with outpatient antibiotics

A. Admission to the floor and start empiric antibiotics

A 54-year-old man presents to the office for a routine visit. He is in otherwise good health and does not take any medications. He notes that over the past few months, he has been experiencing more fatigue and a decreased appetite. On physical exam, his temperature is 97.6°F, blood pressure is 132/74 mm Hg, heart rate is 86 bpm, oxygen saturation is 98% on room air, and respiratory rate is 20/minute. He does not have any evidence of pitting edema or skin turgor. His laboratory results are as follows: a sodium level of 127 mEq/L, a potassium level of 4.6 mEq/L, a serum osmolality of 214 mOsm/kg, a serum uric acid of 1.5 mg/dL, and a thyroid-stimulating hormone level of 2.7 mU/L. Which of the following is the best initial management of this condition? A. Advise fluid restriction B. Assure no further therapy is needed C. Increase free water intake D. Initiate loop diuretics

A. Advise fluid restriction

A patient recently diagnosed with rheumatoid arthritis presents to the clinic for a follow-up visit. She was originally placed on steroids for acute anti-inflammatory benefit and then was bridged to the standard first-line disease-modifying antirheumatic agent. Which of the following should be evaluated prior to initiating this medication and during ongoing use? A. Aminotransferases B. Hemoglobin A1C C. Lipid profile D. Ocular exam every year

A. Aminotransferases

A 50-year-old man presents to the clinic for a follow-up of elevated blood pressure. He has a 10 pack-year smoking history and history of myocardial infarction that required one stent in the left anterior descending artery 2 years prior. He was seen 2 weeks ago and had an in-office blood pressure of 150/90 mm Hg. He bought a cuff and measured his blood pressure every day at home. His systolic pressure ranged from 135-139 mm Hg, and his diastolic pressure ranged from 83-86 mm Hg. He reports no headaches, chest pain, or visual changes. A basic metabolic panel obtained at his last visit showed normal results. The patient is Black. What is the most appropriate first-line pharmacologic therapy according to the ACC/AHA hypertension guidelines? A. Amlodipine B. Hydralazine C. Lisinopril D. Metoprolol

A. Amlodipine

A 65-year-old man presents to the internal medicine clinic for routine management of his chronic medical conditions. His medical history includes end-stage kidney disease, hypertension, and hyperlipidemia. He has adhered with his medications and hemodialysis sessions three times per week. He reports fatigue, and otherwise, he has no concerns during today's examination. Vital signs include temperature of 98.8°F, heart rate of 86 bpm, blood pressure of 155/85 mm Hg, respiratory rate of 14 breaths/minute, and oxygen saturation of 96% on room air. Results from a CBC and iron studies are as follows: WBC: 5.5 × 109/µL RBC: 5.1 × 106/µL Hemoglobin: 10.5 g/dL Hematocrit: 46% Platelets: 225 × 109/L Reticulocyte: 0.4% Mean corpuscular volume: 82 fL Red blood cell distribution width: 11.8% Total Iron: 50 µg/dL Total iron-binding capacity: 150 mcg/dL Ferritin: 320 ng/mL Transferrin saturation: 18% Transferrin: 300 mg/dL Peripheral blood smear: red blood cells are of grossly uniform shape and size, though smaller and paler than typical. No evidence of target cells or hemolysis. Which of the following is the most likely explanation for these findings? A. Anemia of chronic disease B. Folate defici

A. Anemia of chronic disease

A 52-year-old woman presents to the emergency department with abdominal pain for the past day. The pain began gradually and has been steadily worsening. She reports associated nausea and vomiting that began a few hours ago. Her medical history is benign. Her vital signs are heart rate of 112 bpm, blood pressure of 121/76 mm Hg, temperature of 101.3°F, and oxygen saturation of 98%. Physical examination reveals scleral icterus and tenderness to palpation of the right upper quadrant. An abdominal ultrasound is ordered and demonstrates intrahepatic biliary dilation. Which of the following is the most appropriate clinical intervention? A. Antibiotics and endoscopic retrograde cholangiopancreatography B. Antibiotics and laparoscopic cholecystectomy C. Intravenous fluids and nothing by mouth D. Percutaneous transhepatic cholangiography and sphincterotomy

A. Antibiotics and endoscopic retrograde cholangiopancreatography

A 40-year-old man presents to the clinic for progressively worsening abdominal pain over the last year. He describes the pain as cramping, and he is also experiencing intermittent diarrhea. He has noticed increased fatigue, joint pain, and weight loss over the last several months. His vital signs are a blood pressure of 122/82 mm Hg, a heart rate of 82 bpm, a respiratory rate of 14/min, oxygen saturation of 96%, and a temperature of 99.1°F. Physical examination reveals lungs that are clear to auscultation, and heart examination shows a regular rate and rhythm. Upon examination of the oropharynx, ulcerations are noted around the gingivae. Abdominal examination reveals mild diffuse abdominal tenderness with normal bowel sounds. The remainder of the physical examination is within normal limits. Laboratory studies are obtained and reveal a white blood cell count of 11,000/µL, a hemoglobin of 12.2 g/dL, and a C-reactive protein of 7 mg/dL. A colonoscopy is performed, which reveals focal ulcerations next to normal mucosa and a cobblestone appearance to the affected mucosa. Which of the following is true for the most likely diagnosis? A. Associated abscesses and anal fissures are common B. As

A. Associated abscesses and anal fissures are common

A 40-year-old woman presents to the clinic for concerns of cough and wheezing with exercise. Her symptoms have been present for about 2 months. She has no chest pain but describes some chest tightness. Symptoms last for about 1 hour after exercise. She has a history of seasonal allergies and currently smokes cigarettes. The patient appears well on exam. Vital signs show a heart rate of 85 bpm, respiratory rate of 12/minute, oxygen saturation of 98% on room air, blood pressure of 114/76 mm Hg, and temperature of 97°F. On the physical exam, she has a normal heart rate and rhythm with no murmurs, and lungs are clear to auscultation with no wheezes or rhonchi and a normal expiratory phase. You order a pulmonary function test. The patient's FEV1 is 2 L, and her FVC is 3 L. The FEV1/FVC is 0.66. After bronchodilator administration, her FEV1 is 3.25 L, and her FVC is 3.5 L. Her FEV1/FVC is 0.93. Based on these findings, what is the most likely diagnosis? A. Asthma B. Chronic bronchitis C. Emphysema D. Interstitial lung disease

A. Asthma

A 32-year-old man presents with a chronic cough that has been productive of copious amounts of purulent, foul-smelling sputum for the last several weeks. In addition, he reports a few episodes of blood-tinged sputum. He reports no fever, recent illness, or chest pain. His medical history is significant for alpha-1 antitrypsin deficiency and tobacco use. He requires supplemental oxygen continuously at 2 L via nasal cannula. Vital signs are a heart rate of 84 bpm, blood pressure of 124/68 mm Hg, temperature of 97.9°F, and oxygen saturation of 93%. Auscultation of the lungs reveals mild end-expiratory wheezing and crackles heard best at the bases bilaterally. Considering the most likely diagnosis, which of the following radiographic findings would you expect to find on computed tomography of the chest? A. Bronchial dilation with lack of tapering and the signet ring sign B. Honeycombing cysts and reticular septal thickening C. Pulmonary nodules with peripheral ground-glass opacities D. Well-defined nodules with central calcifications

A. Bronchial dilation with lack of tapering and the signet ring sign

A 41-year-old woman presents to the office for evaluation of headaches. Headaches have been occurring off and on for the past 5 years. There is no light sensitivity, sound sensitivity, nausea, or vomiting. She reports her last menstrual period was over 6 years ago before becoming pregnant with her youngest child. Her medical history is pertinent for possible infertility, although the patient reports she has not had a thorough workup despite wanting more children. She takes no medications. Her vital signs reveal a blood pressure of 118/74 mm Hg, heart rate of 72 beats per minute, respiratory rate of 12 breaths per minute, and oxygen saturation of 99%. The patient is afebrile. Breast examination reveals milky nipple discharge on bilateral palpation of the nipples. Neurologic examination reveals a mild deficit within the bilateral temporal visual fields but is otherwise unremarkable. An MRI of the brain is ordered and included above. What is the preferred initial management for the patient's suspected condition? A. Cabergoline B. Octreotide C. Spironolactone D. Transsphenoidal surgery

A. Cabergoline

A 20-year-old man presents to the clinic with right knee pain for the past 2 days. The patient reports no traumatic injury to the knee and no other joint pain. He reports that 2 weeks ago, he had nausea, vomiting, and diarrhea that resolved without treatment. Vital signs include a BP of 120/80 mm Hg, HR of 80 bpm, RR of 20/minute, T of 98.6°F, and SpO2 of 99% on room air. Physical examination findings include generalized right knee tenderness with an effusion and warmth. The patient has full range of motion in the right knee and is able to ambulate. The abdomen is soft and nontender. The patient undergoes an arthrocentesis with synovial fluid analysis, which shows no crystals. The culture grows no organisms. Which of the following organisms is most often associated with the suspected condition? A. Campylobacter jejuni B. Escherichia coli C. Staphylococcus aureus D. Treponema pallidum

A. Campylobacter jejuni

A 24-year-old man is admitted to the hospital for chills, dysuria, and scrotal pain for 3 days. He has no significant medical history. He reports being sexually active with multiple partners. Vital signs today include a heart rate of 105 bpm, blood pressure of 128/86 mm Hg, respiratory rate of 18/minute, oxygen saturation of 97% on room air, and temperature of 101.2°F. Physical examination reveals tachycardia with normal heart rhythm, clear lungs bilaterally on auscultation, soft and nontender abdomen on palpation, tender left testicle on palpation with relief of pain when the scrotum is lifted, minimal scrotal swelling, and intact cremasteric reflex. What is the treatment of choice for the suspected diagnosis? A. Ceftriaxone plus doxycycline B. Emergent urology referral C. Levofloxacin D. Nitrofurantoin

A. Ceftriaxone plus doxycycline

A 44-year-old man with a medical history of hypertension and hyperlipidemia presents to a local urgent care clinic with left foot pain and swelling for the past 2 days. He was recently started on new medications to treat his elevated lipids and elevated blood pressure. Vital signs today include a heart rate of 87 bpm, blood pressure of 137/82 mm Hg, respiratory rate of 20/minute, oxygen saturation of 99% on room air, and temperature of 98.9°F. Physical examination reveals a regular rate and rhythm, no stridor or signs of respiratory distress, no abdominal tenderness to palpation, and a warm, erythematous, and swollen left toe that is painful to palpation. An X-ray of the patient's left foot is shown above. Which of the following medications is likely contributing to his symptoms? A. Chlorthalidone B. Losartan C. Metoprolol D. Pravastatin

A. Chlorthalidone

A 72-year-old man with a medical history of hypertension treated with lisinopril presents to his primary care clinic for a 6-month follow-up after starting a new medication to prevent progression of benign prostatic hyperplasia. Vital signs today include a heart rate of 67 bpm, blood pressure of 129/77 mm Hg, respiratory rate of 19/minute, oxygen saturation of 96% on room air, and temperature of 98.4°F. Physical examination reveals a regular rate and rhythm, no stridor or signs of respiratory distress, normal bowel sounds, and no suprapubic tenderness to palpation. Which of the following describes the most common side effect of this new medication? A. Decreased libido B. Floppy iris syndrome C. Hearing loss D. Orthostatic hypotension

A. Decreased libido

An 81-year-old man admitted to the hospital 2 days ago with pneumonia is evaluated on early morning rounds. According to the night team, the patient did not sleep well and appeared to be conversing with people in the room who were not really present. His medical history is pertinent for atrial fibrillation and hyperlipidemia, for which he takes apixaban and atorvastatin. He is receiving ertapenem and doxycycline for the pneumonia. His vital signs reveal a blood pressure of 152/98 mm Hg, heart rate of 86 beats per minute, respiratory rate of 24 breaths per minute, and oxygen saturation of 95% on a 2 L nasal cannula. He is afebrile. On examination, he appears confused and is frequently distracted. His neurologic examination reveals no focal deficits. CT scan of the brain demonstrates age-related atrophy but is unremarkable. What is the most likely diagnosis? A. Delirium B. Dementia C. Stroke D. Sundowning

A. Delirium

A 66-year-old man presents to the clinic reporting worsening malaise, memory loss, and tingling sensation of his bilateral legs over the last 6 months. His medical history includes type 2 diabetes treated with metformin 1,000 mg twice daily for the past 5 years. Vital signs include a heart rate of 65 bpm, blood pressure of 110/70 mm Hg, temperature of 98.6°F, RR of 16/min, and oxygen saturation of 98% on room air. His physical exam findings reveal a hyperpigmented swollen tongue. Laboratory findings reveal a hemoglobin of 10.5 g/dL, a mean corpuscular volume of 116 fL, and a peripheral smear shown above. Which of the following laboratory findings are most consistent with the diagnosis? A. Elevated methylmalonic acid level and elevated homocysteine level B. Elevated methylmalonic acid level and normal homocysteine level C. Normal methylmalonic acid level and elevated homocysteine level D. Reduced methylmalonic acid level and reduced homocysteine level

A. Elevated methylmalonic acid level and elevated homocysteine level

A 40-year-old man presents to the clinic reporting abdominal pain and bloating of his upper abdomen, gastric reflux, and diarrhea that is worse after eating for the past 5 years. He mentions that he typically takes over-the-counter omeprazole 20 mg and it usually resolves his gastric reflux and diarrhea symptoms. He is worried because over the last 3 months the symptoms have worsened despite use of the proton pump inhibitor, and he has unintentionally lost 10 lbs. He mentions that his father has a history of peptic ulcer disease. Vital signs include a heart rate of 80 bpm, blood pressure of 120/78 mm Hg, temperature of 98.6°F, RR of 16/min, and oxygen saturation of 99% on room air. He undergoes endoscopy, which reveals enlarged gastric folds, esophagitis, and multiple duodenal ulcers. Which of the following laboratory testings is the most appropriate initial screening test to order to help diagnose his condition? A. Fasting serum gastrin concentration B. Fasting serum glucagon concentration C. Gastric acid secretion study D. Secretin stimulation test

A. Fasting serum gastrin concentration

A 40-year-old woman presents to the clinic for follow-up after having new-onset hypertension 2 months ago. She reports mild and intermittent generalized headaches when her blood pressure is high but no chest pain or shortness of breath. She has previously been seen every 6 months for management of her hypothyroidism and has always had normal blood pressures. The only medications she takes are levothyroxine for hypothyroidism and lisinopril for hypertension. Vital signs include a BP of 160/110 mm Hg, HR of 80 bpm, RR of 20/minute, T of 98.6°F, and SpO2 of 98% on room air. Her body mass index is 23.5 kg/m2. Physical examination reveals a regular rate and rhythm and lungs that are clear to auscultation bilaterally. Her abdomen is soft and nontender without palpable masses. Laboratory studies include a creatinine of 1.8 mg/dL and thyroid-stimulating hormone of 1.5 mU/L. CT angiography of the renal arteries reveals a beaded appearance of the right renal artery. Which of the following is the most likely cause of the patient's symptoms? A. Fibromuscular dysplasia B. Pheochromocytoma C. Polycystic kidney disease D. Renal artery stenosis from atherosclerosis

A. Fibromuscular dysplasia

A 54-year-old man presents to the office for evaluation of progressively worsening headaches ongoing for the past 6 weeks. The headaches often awake him from sleep. He is having trouble keeping up at work and experiencing word-finding difficulty. His medical history is pertinent for hypertension treated with lisinopril. Vital signs include a blood pressure of 136/84 mm Hg, heart rate of 86 beats per minute, respiratory rate of 14 breaths per minute, and oxygen saturation of 98%. He is afebrile. On physical examination, he has right-sided hyperreflexia and a right grasping reflex. Papilledema is noted on funduscopic examination. MRI of the brain is ordered and included above. What is the most likely diagnosis? A. Glioblastoma multiforme B. Hypertensive emergency C. Meningioma D. Neurocysticercosis

A. Glioblastoma multiforme

A 56-year-old man presents to the office with increasing fatigue, anxiety, and muscle cramping that began after he had a total thyroidectomy 2 weeks ago. He also reports numbness and tingling in his hands and feet. On physical exam, his temperature is 98.1°F, blood pressure is 128/72 mm Hg, heart rate is 92 bpm, oxygen saturation is 98% on room air, and respiratory rate is 22/minute. Tapping his facial nerve causes contraction of the facial muscles on the same side. His deep tendon reflexes are also hyperactive. Which of the following laboratory findings would most likely be consistent with this patient's condition? A. Hyperphosphatemia, hypocalcemia, decreased parathyroid hormone level B. Hyperphosphatemia, hypocalcemia, elevated parathyroid hormone level C. Hypophosphatemia, hypercalcemia, decreased parathyroid hormone level D. Hypophosphatemia, hypercalcemia, elevated parathyroid hormone level

A. Hyperphosphatemia, hypocalcemia, decreased parathyroid hormone level

A 67-year-old woman with a history of right lower extremity deep vein thrombosis, hyperlipidemia, and breast cancer presents to the clinic with right lower extremity pain. The patient reports aching pain in her right lower extremity that has been gradually worsening over the past year. She describes the pain as a heavy and burning sensation. She states the pain is worse when standing or sitting for long periods of time and is relieved with walking. She also reports noticing swelling of the right lower extremity that gets worse at night. The patient takes atorvastatin 20 mg daily and anastrozole 1 mg daily. Her vital signs are a heart rate of 78 bpm, blood pressure of 128/83 mm Hg, respiratory rate of 16/minute, oxygen saturation of 98% on room air, and temperature of 98.4°F. Which of the following physical exam findings is associated with this patient's condition? A. Lipodermatosclerosis B. Shiny, hairless skin C. Ulceration at the lateral malleolus D. Xeroderma pigmentosum

A. Lipodermatosclerosis

A 45-year-old woman presents to the hepatologist for a follow-up. She has no concerns at this time and is feeling well. Her medical history includes acute hepatitis B infection 3 years ago. What laboratory findings would suggest hepatitis B immunity due to prior resolved infection? A. Positive hepatitis B surface antibody and IgG hepatitis B core antibody B. Positive hepatitis B surface antibody only C. Positive hepatitis B surface antigen and IgG hepatitis B core antibody D. Positive hepatitis B surface antigen and IgM hepatitis B core antibody

A. Positive hepatitis B surface antibody and IgG hepatitis B core antibody

A 78-year-old woman is admitted to the hospital for generalized weakness, fever, and vomiting. She lives at a long-term care facility, and the staff report she has had urinary frequency and foul-smelling urine for several days. Vital signs include a temperature of 101.7°F, heart rate of 106 bpm, blood pressure of 100/74 mm Hg, respiratory rate of 22/minute, and oxygen saturation of 96% on room air. Complete blood count reveals a white blood cell count of 17,000/µL. Which of the following is the most appropriate initial management for the suspected diagnosis? AAdminister broad-spectrum antibiotics and perform intravenous fluid resuscitation BObtain immediate head CT CStart vasopressors DTransfuse two units of packed red blood cells

AAdminister broad-spectrum antibiotics and perform intravenous fluid resuscitation

A 70-year-old man presents to the clinic for his annual visit. He has no concerns today but is curious about possible adverse effects of his medications. His medical history is significant for hypertension, type 2 diabetes mellitus, and heart failure. His conditions are well controlled on lisinopril, hydrochlorothiazide, atenolol, and metformin. Vital signs are T 98.6°F, BP 124/88 mm Hg, HR 64 bpm, RR 14 breaths/min, and SpO2 99% on room air. Which of the following symptoms can be caused by atenolol? A. Dizziness and flushing B. Dysgeusia and cough C. Fatigue and sleep disturbance D. Hypokalemia and hyperuricemia

C. Fatigue and sleep disturbance

Which of the following is a parameter for diagnosis of orthostatic hypotension? A. A 10 mm Hg decrease in systolic blood pressure after moving from a supine to standing position B. A 20 mm Hg decrease in systolic blood pressure after moving from a supine to standing position C. A decrease in heart rate > 10 bpm upon standing D. An increase in heart rate > 30 bpm upon standing

B. A 20 mm Hg decrease in systolic blood pressure after moving from a supine to standing position

A 56-year-old man presents to the internal medicine clinic to establish care. He has not seen a primary care clinician since college and takes no medications daily. He reports gradual onset shortness of breath and decreased activity tolerance over the last 12 months that most bothersome during his 10-hour workdays as a construction worker. He reports no fever, chills, cough, chest pain, nausea or vomiting, or history of venous thromboembolism. The patient lives alone, reports no tobacco or substance use, and drinks six beers per night. A chest radiograph demonstrates cardiomegaly, and an echocardiogram shows left ventricular dilation, left atrial enlargement, and an ejection fraction of 40%. Which clinical exam finding is most consistent with the likely diagnosis? A. Ascites B. Atrial fibrillation C. Distended jugular veins D. Pulsus paradoxus

B. Atrial fibrillation

A 62-year-old woman with a history of depression on amitriptyline 25 mg once nightly for a month presents to the clinic with acute severe right eye pain, nausea, and vomiting. She mentions she woke up this morning with these symptoms and started seeing an appearance of a ring around bright lights. Vital signs include a heart rate of 90 bpm, blood pressure of 125/89 mm Hg, temperature of 98.6°F, RR of 20/min, and oxygen saturation of 99% on room air. Her visual acuity of her right eye is 20/100 and her left eye is 20/40. Her physical exam shows right conjunctival redness, conjunctival vessels injected, cloudy cornea, and a mid-dilated pupil that does not react to light. She is referred to her local ophthalmologist who is willing to see her for a same-day appointment. Which of the following medications is contraindicated in her treatment plan? A. Acetazolamide B. Atropine C. Pilocarpine D. Timolol

B. Atropine

A 55-year-old woman presents to the office with three days of headache, double vision, and pain with chewing. Physical examination reveals moderate tenderness to palpation over the temporal area on the right side. A blood test is taken, which reveals an erythrocyte sedimentation rate of 60 mm/h. Which of the following diseases is often associated with this condition? A. Fibromyalgia B. Polymyalgia rheumatica C. Polymyositis D. Rheumatoid arthritis

B. Polymyalgia rheumatica

An 18-year-old woman presents to the clinic with severe pain in her spine and legs for the past 4 hours. She has a medical history of avascular necrosis of the right knee and has been experiencing bouts of severe whole-body pain since she was 6 months old. Vital signs today include a heart rate of 100 bpm, blood pressure of 130/90 mm Hg, respiratory rate of 20/minute, oxygen saturation of 98% on room air, and temperature of 100°F. Physical exam reveals jaundice and bilateral ankle ulcers. Laboratory investigations reveal a hemoglobin of 9 g/dL, hematocrit of 20%, and reticulocytes of 6%. What is the dose-limiting side effect of the preventative medication recommended for this patient's suspected diagnosis? A. Acute kidney injury B. Bone marrow suppression C. Liver function test abnormalities D. QT prolongation

B. Bone marrow suppression

A 25-year-old woman with no significant medical history presents to the internal medicine clinic for evaluation of a rash in her axillary region for the past week. She recently returned from a hiking trip in western North Carolina and notes the rash has been getting larger over the past few days. The lesion is slightly raised, erythematous, and has an area with central clearing. The patient also reports mild headache and fatigue and reports no associated pruritus or drainage. Vital signs are temperature of 99.2°F, heart rate of 82 bpm, blood pressure of 118/78 mm Hg, respiratory rate of 12 breaths/minute, and oxygen saturation of 99% on room air. Which pathogen is the causative organism of the most likely diagnosis? A. Babesia microti B. Borrelia burgdorferi C. Ixodidae scapularis D. Rickettsia rickettsii

B. Borrelia burgdorferi

A 42-year-old man presents to the emergency department with headache, fever, body aches, and hand weakness for the past 3 days. His medical history includes HIV, which was diagnosed 10 years ago. He had been taking antiretroviral therapy but has not had access to the medication for the past year. Vital signs include a heart rate of 98 bpm, blood pressure of 140/88 mm Hg, respiratory rate of 22/minute, oxygen saturation of 97% on room air, and a temperature of 102.3°F Physical examination reveals mild mental status changes, focal weakness, and slurred speech. His CD4 count is 90 cells/microL. Evaluation of CSF reveals a white blood cell count of 10 cells/µL and protein of 85 mg/dL. All other CSF testing is normal. The patient has an MRI of the brain that is shown above. Which of the following is the most likely diagnosis? A. Central nervous system lymphoma B. Cerebral toxoplasmosis C. Cryptococcal meningitis D. Herpes simplex encephalitis

B. Cerebral toxoplasmosis

A 56-year-old man presents to the office with exertional dyspnea and fatigue that has been worsening over the last month. His medical history is unremarkable, and he does not take any prescription medications or smoke. His vital signs are a heart rate of 78 bpm, respiratory rate of 16/min, temperature of 98.7°F, and blood pressure of 148/62 mm Hg. Physical exam reveals a caudally and laterally displaced apical impulse. Auscultation of the heart sounds reveals a soft, high-pitched diastolic decrescendo murmur best heard in the third intercostal space along the left sternal border. Which of the following is the most likely cause of these findings? A. Aortic dissection B. Congenital bicuspid aortic valve C. Infective endocarditis D. Trauma

B. Congenital bicuspid aortic valve

A 62-year-old man presents to the office for a routine office visit. He recently had a left thyroidectomy and has since been feeling more fatigued than usual, with more muscle cramping in his arms and legs and numbness around his mouth and fingertips. On physical exam, his temperature is 98.6°F, blood pressure is 112/64 mm Hg, heart rate is 84 bpm, oxygen saturation is 98% on room air, and respiratory rate is 18/minute. Laboratory results are as follows: a sodium level of 140 mEq/L, a potassium level of 4.2 mEq/L, an ionized calcium level of 3.5 mg/dL, an albumin level of 4.0 g/dL, and a thyroid-stimulating hormone level of 3.0 mU/L. Which of the following physical exam findings are most likely to be present in a patient with this condition? A. Band keratopathy B. Contraction of facial muscles by tapping the facial nerve C. Decreased muscle strength in the extremities D. Diminished deep tendon reflexes

B. Contraction of facial muscles by tapping the facial nerve

A 42-year-old woman is admitted to the intensive care unit for a subarachnoid hemorrhage. Her medical history includes hypertension and tobacco dependence. She is not on any home medications. During hospitalization, her condition abruptly deteriorates. Vital signs reveal a BP of 156/88 mm Hg, HR of 52 bpm, SpO2 of 94%, and RR of 26/min. She is afebrile. Her breathing pattern is irregular, alternating between periods of rapid shallow breathing and slow deep breaths. She is nonresponsive to verbal stimuli. Her eyes are deviated downward, and her pupils are small and slow to react. Upon applying a painful stimulus, her arms flex inward with fists clench. How would this physical examination finding be documented? A. Decerebrate posturing B. Decorticate posturing C. Hemiplegic posturing D. Opisthotonic posturing

B. Decorticate posturing

A 64-year-old man presents with worsening dyspnea on exertion and a nonproductive cough over the past several months. His social history is significant for tobacco use at one pack per day for 34 years. On physical examination, bibasilar crackles are auscultated. No other concerning physical findings are present. Vital signs are a heart rate of 76 bpm, blood pressure of 132/76 mm Hg, temperature of 98.9°F, and oxygen saturation of 94%. A computed tomography scan of the chest is completed and demonstrates peripheral, basilar opacities with honeycombing and traction bronchiectasis. Which of the following findings would be expected with spirometry testing? A. Decreased FEV1/FVC B. Decreased FVC C. Increased residual volume D. Increased tidal volume

B. Decreased FVC

A 26-year-old woman presents to the clinic with a concern of mild jaundice without any other symptoms. She has no significant medical history, and her only medication is an oral contraceptive. Vital signs include a heart rate of 80 bpm, blood pressure of 112/68 mm Hg, respiratory rate of 20/minute, oxygen saturation of 99% on room air, and temperature of 98.4°F. The physical exam demonstrates mild icterus but otherwise is unremarkable. Laboratory testing is performed, and the CBC, serum albumin, cholesterol, alanine and aspartate aminotransferases, alkaline phosphatase, and prothrombin time are all within normal limits. The total bilirubin is 4.1 mg/dL and direct bilirubin is 2.0 mg/dL. Which of the following is the most likely diagnosis? A. Crigler-Najjar syndrome B. Dubin-Johnson syndrome C. Gilbert syndrome D. Viral hepatitis

B. Dubin-Johnson syndrome

A 74-year-old man presents with fatigue, increasing dyspnea, and a productive cough with pink, frothy sputum for 2 days. He has a medical history of hypertension controlled on losartan for 15 years. He has a 15 pack-year smoking history and quit smoking 6 months ago. Vital signs are T 98.8°F, BP 144/84 mm Hg, HR 90 bpm, RR 22 breaths/min, and SpO2 92% on room air. Physical exam reveals peripheral edema, jugular venous distension, and bilateral rales on auscultation. What diagnostic study result distinguishes a diagnosis of systolic heart failure from diastolic heart failure? A. B-type natriuretic peptide of 85 pg/mL B. Ejection fraction of < 40% C. Evidence of Q waves on ECG D. Pulmonary congestion on chest X-ray

B. Ejection fraction of < 40%

A 72-year-old man with a history of diabetes, hypertension, and hyperlipidemia is hospitalized for abnormal laboratory values and increased confusion. His home medications include metformin, glipizide, amlodipine, and atorvastatin. On admission, he states that his primary care physician sent him to the emergency department after drawing routine labs the day prior. He cannot recall what the labs were drawn for or which labs were abnormal. On review of systems, he reports fatigue, generalized weakness, nausea, vomiting, and weight loss. He also reports mid-back pain, which he attributes to a fall 6 months ago at home. Vital signs include a pulse of 95 bpm, blood pressure of 120/80 mm Hg, respirations of 14 breaths/minute, and oxygen saturation of 98% on room air. Physical examination reveals a thin man in no acute distress. He is alert and oriented to self, but he is not able to state his location or the current year. The remainder of his exam, including cardiac, pulmonary, abdominal, and neurologic exams, is normal. A CT scan of the entire spine is obtained and shows widespread lytic lesions with loss of height. Which of the following laboratory findings would you expect based on the most

B. Elevated creatinine

A 75-year-old woman with a medical history of hypertension, diabetes mellitus, and tobacco use presents to the emergency department for evaluation of chest pain and nausea. Workup reveals an anterior ST elevated myocardial infarction, for which she is taken to the cardiac perfusion lab. Upon evaluation on hospital day #2, the patient is agitated and reports new-onset chest pain and nausea. Vital signs demonstrate temperature of 98.8°F, heart rate of 125 bpm, blood pressure of 88/65 mm Hg, respiratory rate of 24 breaths/minute, and oxygen saturation of 86% on room air. A bedside stat ultrasound is shown above. This presentation is most consistent with which diagnosis? A. Dressler syndrome B. Free wall rupture C. Papillary muscle rupture D. Ventricular aneurysm

B. Free wall rupture

A 42-year-old man presents to the clinic complaining of a painful, swollen elbow joint. He states his symptoms were present when he awoke this morning. His past medical history includes diabetes mellitus and hypertension, for which he takes metformin and hydrochlorothiazide. Physical exam findings reveal fever and a tender, warm, erythematous elbow joint. Laboratory analysis of aspirated synovial joint fluid reveals monosodium urate crystals and an elevated white blood cell count. Which of the following diagnoses are consistent with this result? A. Basic calcium phosphate crystal disease B. Gout C. Pseudogout D. Septic arthritis

B. Gout

A 30-year-old man with no significant past medical history presents to the office complaining of a persistent, productive cough for the past week. On examination, his temperature is 99.5°F, and his lungs are clear to auscultation. What is the best treatment for this patient? A. Azithromycin B. Guaifenesin C. Levofloxacin D. Oseltamivir

B. Guaifenesin

A 46-year-old man with a history of gastric ulcer treated with antacids presents to the gastroenterology clinic with weight loss, early satiety, and persistent epigastric abdominal pain for 2 months. He also notes some recent black tarry stools. Vital signs include a heart rate of 82 bpm, blood pressure of 132/78 mm Hg, respiratory rate of 20/minute, oxygen saturation of 100% on room air, and temperature of 98.6°F. Physical examination shows no palpable masses or tenderness of the abdomen but does reveal a palpable left supraclavicular lymph node. What is the most common risk factor for this condition? A. Alcohol consumption B. Helicobacter pylori infection C. High intake of dietary fiber D. NSAID use

B. Helicobacter pylori infection

A 60-year-old man presents to your clinic for a new-patient visit. He has not been seen by a physician in over 10 years and has a 20 pack-year smoking history. He states he has had a worsening, persistent, productive cough for the past 4 months. He also notes he becomes more easily short of breath with activity than before. He reports no chest pain, lower extremity swelling, orthopnea, or paroxysmal nocturnal dyspnea. His vital signs show a heart rate of 89 bpm, oxygen saturation of 94% on room air, temperature of 97°F, respiratory rate of 14/minute, and blood pressure of 125/88 mm Hg. On exam, he appears comfortable and in no acute distress. His cardiac exam reveals normal cardiac rate and rhythm without murmurs, 2+ distal pedal and radial pulses, and no lower extremity edema. Respiratory exam reveals no wheezes or rhonchi but a prolonged expiratory phase. His chest X-ray is shown above. What other physical exam finding would be expected based on the suspected diagnosis? A. Dullness to percussion B. Hyperresonance to percussion C. Increased tactile fremitus D. Trachea shifted toward the left side

B. Hyperresonance to percussion

A registered nurse with no significant past medical history is being evaluated for tuberculosis after she recently cared for a patient who tested positive for active tuberculosis. She reports she didn't always use correct infectious disease precautions when she entered the patient's room. She does not have any current respiratory or constitutional symptoms. Which of the following is required to make a definitive diagnosis of tuberculosis in a health care worker? A. Chest X-ray showing Ranke complexes B. Identification of Mycobacterium tuberculosis on culture C. Presence of acid-fast bacilli on sputum sample D. Tuberculin skin test reaction size of 11 mm

B. Identification of Mycobacterium tuberculosis on culture

A 24-year-old man presents to the clinic with a dull aching sensation on the left side of his scrotum for the past month. He reports that the symptoms improve when he is lying down. Vital signs include a BP of 120/80 mm Hg, HR of 80 bpm, RR of 20/minute, T of 98.6°F, and SpO2 of 99% on room air. Physical examination reveals a regular heart rate and rhythm and a soft, nontender abdomen. The patient has a left-sided scrotal mass that is soft and nontender. He has no intratesticular tenderness or masses. Which of the following is a complication of the suspected condition? A. Epididymitis B. Infertility C. Intestinal obstruction D. Testicular torsion

B. Infertility

A 27-year-old woman with a history of systemic lupus erythematosus presents to the clinic. Her current medications include hydroxychloroquine and methotrexate. She states that she has noticed increased bruising and minor gingival bleeding over the past 2 weeks. She reports no recent trauma or falls. Her vitals include a temperature of 37°C, pulse of 85 bpm, blood pressure of 115/75 mm Hg, respirations of 14 breaths/minute, and oxygen saturation of 99% on room air. Physical exam reveals multiple scattered ecchymoses to bilateral arms and legs, petechial rash to the anterior aspects of bilateral legs, and purpuric oropharyngeal mucous membranes. The remainder of the physical exam is unremarkable. Laboratory evaluation is significant for a platelet count of 48,000/µL. White blood count, hemoglobin, hematocrit, prothrombin time, and INR are within normal limits. Peripheral blood smear reveals large sized platelets. What is the best next step in treatment of the suspected diagnosis? A. Administer intravenous immune globulin B. Initiate dexamethasone therapy C. Initiate rituximab therapy D. No pharmacological therapy is needed

B. Initiate dexamethasone therapy

A 46-year-old woman with a history of hypothyroidism and generalized anxiety disorder presents to the clinic for evaluation. Her current medications include levothyroxine and escitalopram. She reports progressive fatigue and swelling of her hands and fingers. She states that she is now having trouble clenching her hands into a fist. She has also noticed that her fingers intermittently turn white, especially in cold temperatures. Her vitals include a temperature of 37°C, blood pressure of 120/80 mm Hg, pulse of 75 bpm, respirations of 14 breaths/minute, and oxygen saturation of 98% on room air. On examination, she is sitting comfortably in the clinic chair and is in no acute distress. She has swollen fingers and nonpitting edema of her bilateral hands. There is no lower extremity edema. The skin on her hands and forearms appears tight, shiny, and thickened. She has decreased range of motion in her bilateral fingers and wrists. The remainder of her physical exam is unremarkable. Laboratory evaluation revealed a positive antinuclear antibody. Which of the following is the patient at risk of developing based on the suspected diagnosis? A. Erythema nodosum B. Interstitial lung disease C. Ke

B. Interstitial lung disease

A 48-year-old man is admitted to the hospital due to fever of unknown origin and shortness of breath. His medical history is significant for intravenous drug use. Vital signs are T 38.5°C, BP 110/68 mm Hg, HR 105 bpm, RR 15 breaths/min, and SpO2 97% on room air. Two sets of blood cultures are obtained and are positive for Staphylococcus aureus. Which of the following additional findings would confirm the suspected diagnosis? A. Hematuria B. Janeway lesions C. Leukocytosis D. Nikolsky sign

B. Janeway lesions

A 62-year-old man with alcohol use disorder is brought to the emergency department by his spouse, who states that he appears disoriented and confused. The patient's speech is slurred, and his spouse states that he cannot walk straight. His vital signs are a blood pressure of 128/88 mm Hg, heart rate of 86 bpm, respiratory rate of 18/min, oxygen saturation of 93%, and temperature of 98.4°F. Upon physical examination, the skin appears jaundiced, and the abdomen is mildly distended. Muscle wasting in the upper extremities is also noted. The heart examination is within normal limits, and the lungs are clear on auscultation. Nystagmus is noted upon cranial nerve testing, and patellar deep tendon reflexes are hyperactive. A noncontrast CT of the head is obtained and reveals generalized cerebral edema. Laboratory studies are obtained and reveal an ammonia level of 75 µmol/L, an aspartate aminotransferase level of 65 U/L, an alanine aminotransferase level of 73 U/L, and an alkaline phosphatase level of 150 IU/L. Which medication is the most appropriate treatment for the patient's suspected diagnosis? A. Ceftriaxone B. Lactulose C. Rifaximin D. Thiamine

B. Lactulose

A 55-year-old man with a history of cigarette smoking is admitted to the hospital after presenting with chest pain and dyspnea with exertion. His initial ECG is shown above. His initial troponin I was 5 ng/mL, and later that day, he was taken to the cardiac catheterization lab for percutaneous coronary intervention and has now been started on aspirin and clopidogrel. He currently has no chest pain and is ready for discharge. His latest vital signs are a BP of 130/70 mm Hg, HR of 80 bpm, RR of 20/min, T of 98.6°F, and SpO2 of 99% on room air. Physical examination reveals a regular heart rate and rhythm and lungs clear to auscultation bilaterally. Which additional medications should this patient be prescribed upon discharge? A. Lisinopril and metoprolol B. Lisinopril, metoprolol, and atorvastatin C. Lisinopril, verapamil, and rosuvastatin D. Metoprolol and losartan

B. Lisinopril, metoprolol, and atorvastatin

A 36-year-old woman with a history of systemic lupus erythematosus presents as a new patient to establish care. The patient reports she has had three deep vein thromboses in the last 15 years. She also says she had three miscarriages in the past and she is trying to conceive again. Vital signs are a heart rate of 89 bpm, blood pressure of 126/81 mm Hg, respiratory rate of 16/minute, oxygen saturation of 98% on room air, and temperature of 98.4°F. On physical exam, the patient is alert, attentive, and oriented. The cranial nerves II-XII are intact. Her breath sounds are intact in all lung fields with no rales, wheezes, or rhonchi. The S1 and S2 heart sounds are in regular rhythm and no murmurs are heard. Her lab results are significant for IgG anticardiolipin antibodies. Which of the following physical exam findings is associated with this patient's diagnosis? A. Erythema nodosum B. Livedo reticularis C. Sclerodactyly D. Xerostomia

B. Livedo reticularis

A 35-year-old woman presents to the clinic to discuss cramping abdominal pain for the past 6 months. She states that the abdominal pain is diffuse, occurs once or twice per week, and is associated with diarrhea. Her pain decreases after passing the diarrhea, but she states that her bowels do not feel empty upon defecation. Bowel movements on other days of the week are normal. Vitals signs are a blood pressure of 120/78 mm Hg, heart rate of 80 bpm, respiratory rate of 12/min, oxygen saturation of 99%, and a temperature of 98.6°F. Physical examination reveals normal bowel sounds in all four quadrants and mild diffuse abdominal tenderness upon palpation. Lungs are clear to auscultation, and the heart examination is within normal limits. The remainder of the physical examination is unremarkable. Laboratory studies are unremarkable. What is the best treatment for the management of this patient's condition? A. Linaclotide B. Loperamide C. Lubiprostone D. Polyethylene glycol

B. Loperamide

A 70-year-old man is admitted to the hospital for a cough and shortness of breath that have progressively worsened over the past 3 months. He recently noticed some blood in his sputum. He has a medical history of hypertension, for which he takes amlodipine. He has smoked a pack of cigarettes per day for 40 years. Vital signs include a BP of 130/88 mm Hg, HR of 80 bpm, RR of 24/min, T of 98.6°F, and SpO2 of 93% on room air. Physical examination findings include a regular rate and rhythm with no rubs, gallops, or murmurs. The patient's lungs are clear to auscultation except in the right lower lobe, where he has decreased breath sounds and dullness to percussion. He has no lower extremity peripheral edema or calf tenderness. His chest X-ray is shown above. His urinalysis shows 1+ proteinuria but is otherwise unremarkable. He undergoes a thoracentesis, which shows a pleural fluid protein concentration of 5 g/dL and pleural fluid lactate dehydrogenase of 400 U/L. His serum protein concentration is 7.9 g/dL, and his serum lactate dehydrogenase is 500 U/L. Which of the following is the most likely cause of this patient's symptoms? A. Congestive heart failure B. Lung cancerCorrect Answer C. Nep

B. Lung cancer

A 45-year-old woman presents to her primary care clinician reporting progressive multi-joint pain over the past 3 years. The most notable symptoms are in her hands and digits. She reports that since she turned 40 years old, she has gained weight and noticed increasing multi-joint pain. She assumed once she returned to a healthy weight the joint pain would resolve. Unfortunately, the pain has continued and worsened. She reports some difficulty when she is lifting weights at the gym, but overall, she feels better with movement. She notes increasing stiffness if she does not exercise regularly. In addition to pain, she describes decreased range of motion, swelling, and reduced grip strength. Which of the following additional findings would be most likely in this patient? A. Bony, hard swelling of the distal interphalangeal joints B. Morning stiffness and pain of the joints C. Presence of Heberden nodes D. Tenderness in the distal interphalangeal joints

B. Morning stiffness and pain of the joints

A 32-year-old woman presents to the office for evaluation of numbness of the right arm and face. Symptoms began approximately 1 week ago. She reports no injury, trauma, headache, or difficulty speaking. Her medical history is pertinent for optic neuritis of the right eye occurring 2 years ago, which was treated with a course of intravenous steroids. Vital signs demonstrate a BP of 112/74 mm Hg, HRe of 82 bpm, RR of 16/min, and SpO2 of 99%. She is afebrile. Neurological examination demonstrates decreased sensation within the right upper extremity and right side of the face. Strength is preserved. Hyperreflexia is noted in the right upper extremity. A relative afferent pupillary defect is noted in the right eye. An MRI of the brain is pending. What is the MRI most likely to demonstrate based on the suspected diagnosis? A. Lenticular-shaped collection of blood that does not cross suture lines B. Periventricular and juxtacortical white matter lesions C. Restricted diffusion within left middle cerebral artery territory D. Ventricular enlargement without corresponding cortical atrophy

B. Periventricular and juxtacortical white matter lesions

A 40-year-old woman presents to the clinic to discuss difficulty swallowing, which has become progressively worse over the 2 years. She states that both solid foods and liquids are difficult to swallow, and she will often regurgitate food that is undigested. She has also experienced a burning sensation in her chest after meals. She tried a proton pump inhibitor in the past, but it did not seem to help her symptoms. Her vital signs are a blood pressure of 126/76 mm Hg, a heart rate of 86 bpm, a respiratory rate of 16/min, oxygen saturation of 98%, and a temperature of 98.8°F. Physical examination reveals lungs that are clear to auscultation, and heart examination demonstrates a regular rate and rhythm. The abdomen is soft and nontender, and bowel sounds are noted in all four quadrants. The remainder of the physical examination is within normal limits. Esophageal manometry is obtained and reveals incomplete relaxation of the lower esophageal sphincter and aperistalsis in the distal two-thirds of the esophagus. What is the best treatment for this patient's condition? A. Botulinum toxin injection B. Pneumatic balloon dilation C. Short-acting nifedipine D. Sublingual nitroglycerin before me

B. Pneumatic balloon dilation

A 74-year-old man with a medical history of heart failure, hypertension, hyperlipidemia, and type 2 diabetes mellitus presents to the urgent care clinic with acute left flank pain that began 3 hours ago. He reports taking acetaminophen, which provided minimal relief. The patient reports no symptoms of nausea, vomiting, or dysuria. Vital signs today include a heart rate of 110 bpm, blood pressure of 141/78 mm Hg, respiratory rate of 20/minute, oxygen saturation of 95% on room air, and temperature of 98.4°F. Physical examination reveals a regular rate and rhythm, no stridor or signs of respiratory distress, and left-sided costovertebral angle tenderness. Urinalysis shows RBC > 50 × 106/µL, WBC 5 × 109/L, no bacteria, no nitrites, and a specific gravity of 1.03. A basic metabolic panel shows a creatinine of 3.2 mg/dL, which is at the patient's baseline. A low-radiation-dose computed tomography of the abdomen and pelvis shows a left nonobstructing kidney stone approximately 7 mm in size, with no signs of hydronephrosis. Which of the following describes the most appropriate clinical intervention? A. Prescribe NSAIDs, oral antibiotics, and discharge with close primary care provider follow-up

B. Prescribe NSAIDs, tamsulosin, and discharge with close primary care provider follow-up

A 45-year-old woman presents to the clinic for follow-up. The patient was evaluated in the emergency department last week for an episode of shortness of breath and had a CT scan of her chest to evaluate for a pulmonary embolism. The CT scan did not show a pulmonary embolism but did show a 5 mm solid solitary pulmonary nodule in the right lower part of her lung. The patient has a history of diabetes mellitus, for which she takes metformin and sitagliptin, but has no history of cigarette smoking. Vital signs include HR of 80 bpm, BP of 120/80 mm Hg, RR of 20/min, oxygen saturation of 98% on room air, and T of 98.6°F. Physical examination reveals lungs clear to auscultation bilaterally and no palpable peripheral lymphadenopathy. Which of the following is the recommended next step in management? A. PET scan now to further assess B. Reassurance that the nodule is likely benign and requires no further workup C. Repeat CT scan in 3 months to document stability D. Surgical biopsy of the nodule

B. Reassurance that the nodule is likely benign and requires no further workup

An 81-year-old woman has been admitted to the hospital for observation after a brief episode of chest pain and palpitations. She states she has been having similar episodes intermittently for 1 week. Her medical history includes hypertension and type 2 diabetes mellitus. Current medications include losartan 50 mg daily and metformin 500 mg twice daily. ECG revealed a regular heart rate and rhythm with no ST changes upon arrival to the emergency department. She is on continuous ECG monitoring, and you are notified of the above ECG changes. The patient is reevaluated and reports no chest pain or palpitations. Vital signs at this time include a Glasgow Coma Scale score of 15, heart rate of 148 bpm, blood pressure of 130/92 mm Hg, temperature of 98.6°F, respiratory rate of 18/minute, and oxygen saturation of 96% on room air. Which of the following should be done first for the suspected diagnosis? A. Administer adenosine 6 mg IV B. Administer amiodarone 300 mg IV C. Administer metoprolol 5 mg IV D. Perform synchronized electrical cardioversion

C. Administer metoprolol 5 mg IV

A 65-year-old man presents to the clinic with palpitations intermittently for the past week. He reports having no chest pain or dyspnea. His medical history includes hypertension, for which he takes amlodipine, and hyperlipidemia, for which he takes atorvastatin. Vital signs include a BP of 135/90 mm Hg, HR of 75 bpm, RR of 20/min, T of 98.6°F, and SpO2 of 99% on room air. Physical examination reveals an irregular heart rhythm without murmurs, and his lungs are clear to auscultation bilaterally. His ECG is shown above. Which of the following is the best recommendation regarding stroke prevention in this patient? A. Aspirin alone B. Clopidogrel alone C. No anticoagulation therapy is recommended D. Rivaroxaban

D. Rivaroxaban

A 55-year-old woman presents with multiple concerns. She reports that she has been unable to wear her contact lenses for the last several weeks because of ocular dryness. She has thick, rope-like secretions from her eyes in the mornings and reports a gritty sensation in her eyes that persists constantly. She reports symptoms of a dry mouth as well, stating that she always carries a water bottle with her or else she has difficulty swallowing or speaking because of the extreme dryness. Physical examination reveals mild conjunctival injection and dry mucous membranes. Vital signs are heart rate of 76 bpm, blood pressure of 132/76 mm Hg, temperature of 98.7°F, and oxygen saturation of 98%. Which of the following would most likely be increased considering the most likely diagnosis? A. Anti-double-stranded DNA antibody (anti-DNA antibody) B. Anti-histidyl-transfer RNA synthetase antibody (anti-Jo-1 antibody) C. Anti-Sjögren syndrome A antibody (anti-SS-A antibody) D. Anti-topoisomerase 1 antibody (anti-SCL-70 antibody)

C. Anti-Sjögren syndrome A antibody (anti-SS-A antibody)

A 55-year-old woman presents to the clinic for a follow-up after hospitalization for diverticulitis 6 weeks ago. She recently had a colonoscopy, which revealed moderate diverticula in the colon. Her vital signs are a blood pressure of 122/80 mm Hg, heart rate of 68 bpm, respiratory rate of 16/min, oxygen saturation of 95%, and temperature of 98.8°F. Physical examination reveals a nontender, nondistended abdomen with normal bowel sounds. Lungs are clear to auscultation, and the heart exam is within normal limits. The remainder of the physical examination is unremarkable. Which health maintenance recommendation is most appropriate for this patient? A. Avoidance of nuts and seeds B. Colonoscopy in 6 months C. Consumption of a high-fiber diet D. Limiting physical activity

C. Consumption of a high-fiber diet

A 35-year-old woman is admitted to the hospital after presenting with acute alteration in mental status. She also had been vomiting for the past day. Vital signs include a BP of 160/110 mm Hg, HR of 142 bpm, RR of 22/minute, T of 103.6°F, and SpO2 of 98% on room air. Physical examination findings include a patient who is alert and oriented to person but not place or time. She has no focal neurologic deficits or meningismus. You notice her eyes protrude bilaterally and note lid lag when checking extraocular movements. Lungs are clear to auscultation, and her abdomen is soft and nontender without bruits. Her hands are tremulous. ECG shows sinus tachycardia. The patient's complete blood count reveals a white blood cell count of 12,500 cells/µL. Urinalysis reveals no leukocyte esterase or nitrites, and chest X-ray shows no infiltrates. Which of the following diagnostic study findings are necessary to confirm the most likely diagnosis? A. Elevated T3, elevated T4 two times the upper limit of normal, and low TSH B. Elevated T3, elevated T4, and high TSH C. Elevated T3, elevated T4, and low TSH D. Low T3, low T4, and high TSH

C. Elevated T3, elevated T4, and low TSH

A 68-year-old man has been hospitalized after a non-ST elevation myocardial infarction that occurred 10 hours ago. His medical history includes hypertension and hyperlipidemia. Current medications include enalapril 10 mg daily and atorvastatin 20 mg daily. His initial troponin in the emergency department was elevated to 1.1 ng/mL. Continuous cardiac monitoring begins to alarm, and the above ECG is recorded. The patient is now unresponsive and has no palpable pulse, so cardiopulmonary resuscitation is initiated, beginning with chest compressions. Which of the following therapeutics should be administered first for the suspected diagnosis? A. Adenosine 6 mg IV B. Amiodarone 300 mg IV C. Epinephrine 1 mg IV D. Magnesium sulfate 1 mg IV

C. Epinephrine 1 mg IV

A 68-year-old woman presents to an internal medicine clinic as a new patient. She just moved to the area to live with her children. She reports increasing joint pain and stiffness over the past few years, which she attributes to previously working on a farm and general aging. Her bilateral shoulders and pelvic girdle are most affected. The symptoms are worse in the morning and improve with movement. There is no focal weakness, but she reports malaise and weight loss. She mentions that her last clinician was treating her for anemia. Which of the following laboratory markers is most likely to be abnormal in this patient? A. Antinuclear antibody B. Creatine kinase C. Erythrocyte sedimentation rate D. Rheumatoid factor

C. Erythrocyte sedimentation rate

A 31-year-old woman is admitted to the hospital with respiratory distress. She presented to the emergency department yesterday with new-onset shortness of breath and difficulty handling her secretions. She reports no history of asthma but states that for the past 5 months, she has been experiencing intermittent diplopia, droopy eyelids, and difficulty chewing that generally worsens as it gets later in the day. Her medical history is pertinent for hypothyroidism treated with levothyroxine. Her vital signs include a BP of 114/86 mm Hg, HR of 116 bpm, RR of 26/min, and SpO2 of 95% on high-flow oxygen. She is afebrile. On examination, she is in moderate respiratory distress and accessory muscle use is noted. A neurologic examination is performed, which demonstrates ptosis with sustained upward gaze. Reflexes are 2+ in all four extremities. No muscle atrophy is appreciated. What is the underlying pathophysiology of the patient's suspected condition? A. Degeneration of motor neurons leading to atrophy and gliosis B. Development of reversible airflow obstruction leading to bronchial hyperresponsiveness C. Formation of antibodies against postsynaptic acetylcholine receptors D. Paraneoplastic c

C. Formation of antibodies against postsynaptic acetylcholine receptors

A 76-year-old man presents with worsening dyspnea on exertion, dry cough, and lower extremity edema for the past several weeks. His medical history is significant for COPD, hypertension, and hyperlipidemia. He has a poor diet consisting of mostly processed, high-sodium foods. His social history is significant for tobacco use of one pack per day for 40 years. Vital signs are heart rate of 90 bpm, temperature of 98.5°F, blood pressure of 148/88 mm Hg, and oxygen saturation of 90%. Physical examination reveals nonpitting edema in the lower extremities, hepatomegaly, and jugular venous distention. Cardiac auscultation reveals a right ventricular heave. An echocardiogram is ordered and demonstrates right ventricular hypertrophy and increased tricuspid regurgitant velocity. Which of the following is the most appropriate therapy? A. Calcium channel blocker B. Digoxin C. Furosemide D. Oxygen therapy

C. Furosemide

A 69-year-old woman presents to the office with severe right-sided back pain. The pain has been ongoing for the last 5 months and is described as burning and sharp. She notes that acetaminophen and ibuprofen have done little to control her pain. Her medical history is pertinent for hypertension, glaucoma, and shingles occurring 9 months ago at the right T4 dermatome level. She takes losartan and topical latanoprost. Her vital signs include a blood pressure of 132/84 mm Hg, heart rate of 74 beats per minute, respiratory rate of 18 breaths per minute, and oxygen saturation of 98%. She is afebrile. On physical examination, the skin is without erythema or lesion. The spine and surrounding paraspinal muscles are nontender to palpation and without spasm. There is no costovertebral angle tenderness. What treatment would be appropriate for the patient's pain given the most likely diagnosis? A. Acyclovir B. Amitriptyline C. Gabapentin D. Topical lidocaine

C. Gabapentin

A 44-year-old man presents to the office for pheochromocytoma screening after his brother was recently diagnosed with one. A plasma metanephrine test and computed tomography of the adrenal glands confirm he does have pheochromocytoma. He does not complain of any symptoms. Which of the following signs and symptoms are most commonly associated with this condition? A. Bone pain, kidney stones, nausea, psychosis, and depression B. Dry mouth, decreased sweating, constipation, and urinary retention C. Headaches, palpitations, diaphoresis, and severe hypertension D. Hunger, shakiness, dizziness, trembling, and generalized weakness

C. Headaches, palpitations, diaphoresis, and severe hypertension

A 65-year-old woman with a history of type 2 diabetes mellitus, hypertension, and stage 2 chronic kidney disease presents to the hospital for muscle weakness that began last night. The muscle weakness was initially limited to the lower extremity but rapidly progressed to include the upper extremity. Vital signs today include a heart rate of 45 bpm, blood pressure of 130/90 mm Hg, respiratory rate of 20/minute, oxygen saturation of 98% on room air, and temperature of 98.4°F. Physical exam reveals bradycardia and global muscle weakness. ECG is shown above. Which of the following medications is the most likely to cause this patient's findings? A. Furosemide B. Hydrochlorothiazide C. Ibuprofen D. Metformin

C. Ibuprofen

A 20-year-old man has been hospitalized for generalized abdominal pain, vomiting, hypotension, confusion, and malaise. He also reports 1 month of increased thirst, more frequent urination, and an unintentional 8 lb weight loss. He has no significant medical or surgical history. Vital signs at this time include a temperature of 97.8°F, blood pressure of 100/70 mm Hg, heart rate of 106 bpm, respiratory rate of 32/minute, and oxygen saturation of 97% on room air. Significant physical examination findings include deep, rapid breathing and fruity breath odor. Which of the following lab findings is most consistent with the suspected diagnosis? A. Absence of serum ketones B. Arterial pH of 7.35 C. Serum bicarbonate of 14 mEq/L D. Serum osmolality of 330 mOsm/kg

C. Serum bicarbonate of 14 mEq/L

A 37-year-old man is admitted to the hospital after presenting to the emergency department with a 2-day history of bilateral leg weakness and difficulty walking. Symptoms initially started in the feet but have gradually worsened since onset and now involve the entire legs. He reports no history of injury or trauma. He is relatively healthy but reports a recent bout of food poisoning. He takes no medications. Vital signs include a blood pressure of 108/74 mm Hg, heart rate of 102 beats per minute, respiratory rate of 18 breaths per minute, and oxygen saturations of 97%. He is afebrile. Physical examination demonstrates bilateral lower extremity strength of 3/5 and bilateral upper extremity strength of 4/5. Areflexia is noted throughout. What is the preferred intervention to treat the patient's suspected condition? A. Corticosteroids B. Interferon beta-1a C. Intravenous immunoglobulin D. Pyridostigmine

C. Intravenous immunoglobulin

A 19-year-old man is evaluated in the office for a sick visit. He is accompanied by his college roommate. The patient reports a headache beginning early this morning. It is associated with neck stiffness and light sensitivity. The patient's medical history is pertinent for declined childhood vaccinations. He takes no medications. Vital signs include BP of 102/62 mm Hg, HR of 112 bpm, RR of 18/min, SpO2 of 99%, and T of 101.7°F. On examination, the patient appears lethargic. Bilateral petechiae are noted on the shins. There is significant resistance to knee extension when the hip is flexed at 90 degrees. What physical examination sign does this finding describe? A. Brudzinski sign B. Kehr sign C. Kernig sign D. Lhermitte sign

C. Kernig sign

A 48-year-old man presents to the urology clinic with intermittent urinary frequency and urgency for the past 4 months. He also reports discomfort in his perineal area. The patient reports a remote history of a urethral stricture requiring urologic repair. Vital signs include a BP of 120/80 mm Hg, HR of 80 bpm, RR of 20/minute, T of 99.6°F, and SpO2 of 99% on room air. Physical examination reveals normal external genitalia with no testicular tenderness or masses. His abdomen is soft and nontender with normoactive bowel sounds and no costovertebral angle tenderness. He has mild perineal tenderness, and prostate exam reveals mild tenderness and edema. Urinalysis shows bacteriuria with moderate leukocyte esterase. Which of the following is the most appropriate treatment? A. Ciprofloxacin for 3 weeks B. Doxycycline for 6 weeks C. Levofloxacin for 6 weeks D. Trimethoprim-sulfamethoxazole for 4 weeks

C. Levofloxacin for 6 weeks

A 50-year-old man returns to the family practice clinic to review his lab report from his wellness visit 1 month ago. He works as a truck driver. He has not had a physical in the last 5 years and thus has no established medical history. He is not on any medications and has no history of smoking. He has a family history of diabetes. Vital signs include a heart rate of 80 bpm, blood pressure of 130/85 mm Hg, BMI of 31.0 kg/m2, temperature of 98.6°F, RR of 18/min, and oxygen saturation of 98% on room air. Laboratory findings reveal total cholesterol 205 mg/dL, triglycerides 200 mg/dL, HDL 35 mg/dL, LDL 130 mg/dL, fasting glucose of 124 mg/dL, HbA1C 6.4%, and alanine aminotransferase 40 U/L. Which intervention should be started initially to help manage his disease state? A. Atorvastatin B. Dapagliflozin C. Lifestyle modification D. Metformin

C. Lifestyle modification

A 56-year-old woman presents to the office due to increasing weakness in her arms and legs. It started about 3 months ago and has gotten progressively worse. She states that she has difficulty walking up and down the stairs and getting in and out of a chair. She also noticed that she has been having trouble lifting her groceries. She does not have a cough, wheezing, difficulty swallowing, or chest pain. On physical exam, her T is 98.6°F, BP is 112/64 mm Hg, HR is 84 bpm, oxygen saturation is 98% on room air, and RR is 18/minute. An erythematous butterfly shaped rash is noticeable on her face with nasolabial involvement, and other patches of erythema are present on her neck, both shoulders, and upper anterior chest. There is also a violet colored rash to her bilateral upper eyelids. She does not have any swelling of the lower extremities. This patient is at increased risk for which of the following conditions? A. Emphysema B. Hepatitis B C. Malignancy D. Myasthenia gravis

C. Malignancy

A 23-year-old man presents to the internal medicine clinic as a new patient for evaluation of low back pain that radiates into his buttocks. He reports pain and progressive stiffness for the past 7 years. While he was in college, he worked for a moving company and assumed the back pain was due to the heavy lifting. However, he stopped this job 2 years ago and now works a desk job. He also took up yoga to attempt to relieve the stiffness. His low back pain and stiffness have continued to progress. He reports his symptoms are worse in the morning and improve throughout the day, including with activity. His prior physician had ordered imaging and lab work, which demonstrated a negative rheumatoid factor, negative anti-cyclic citrullinated peptide antibodies, elevated erythrocyte sedimentation rate, and positive human leukocyte antigen B27. A previously obtained X-ray of his lumbar spine demonstrated sclerosis of the attachment of the annulus fibrosus to the vertebral bodies and squaring of the vertebral bodies, and an X-ray of his pelvis demonstrated bilateral erosion and sclerosis of the sacroiliac joints. Which of the following is the recommended first-line treatment for this patient? A.

C. Naproxen

A 66-year-old woman with a 25 pack-year smoking history and a history of hypercholesterolemia, diabetes type 2, and obesity presents to the clinic with left-sided knee pain. The patient reports experiencing a dull pain in her left knee that has been progressively worsening over the past year. She reports the pain is intermittent and notices it is worse first thing in the morning and when walking. The pain covers her entire anterior knee and occasionally radiates down her shin. She reports no trauma to the left knee. The patient reports having eight drinks per week for the past 10 years. She takes atorvastatin 80 mg daily and metformin 1,000 mg twice daily. She eats a diet high in seafood and meats. Her vital signs are a heart rate of 78 bpm, blood pressure of 131/82 mm Hg, respiratory rate of 16/minute, oxygen saturation of 98% on room air, and temperature of 98.4°F. On physical examination, there is no erythema, deformity, swelling, or ecchymosis noted upon inspection of the left knee. There is moderate tenderness to palpation of the joint line of the left knee, the range of motion is slightly limited in bilateral knees, and there is no tendon laxity noted. Radiograph of the left knee i

C. Obesity

A 72-year-old-woman who has recently relocated to the area presents to the clinic to establish care. Her medical history includes hypertension, chronic obstructive pulmonary disease, and rheumatoid arthritis treated with lisinopril, inhaled fluticasone, and methotrexate. While reviewing the patient's history with her, she also tells you that she has been hospitalized for an "intestinal infection" on two separate occasions in the past year. She states that the illness was preceded by antibiotic treatment for other ailments in both instances. Most recently, she was finishing a course of clindamycin prescribed to her for a dental infection when she began to notice frequent watery diarrhea, lower abdominal cramping, and fever. She states that this illness progressed, and she presented to the hospital where she was admitted. She tells you, "They treated me with antibiotics and said my white blood cells and a kidney test were both high." Which of the following is recommended to prevent another such infection? A. Daily probiotic therapy for patients receiving systemic antibiotics B. Oral metronidazole prophylaxis for patients receiving systemic antibiotics C. Oral vancomycin prophylaxis for p

C. Oral vancomycin prophylaxis for patients receiving systemic antibiotics

A 70-year-old man with a medical history of hypertension, hyperlipidemia, and right hip osteoarthritis who underwent a total right hip arthroplasty 4 weeks ago presents to the emergency department with progressive chest pain, dyspnea, and a cough that has developed in the past 2 days. He describes his chest pain occurring when taking in a deep breath and reports dyspnea on exertion. He reports no hemoptysis. Vital signs today include a heart rate of 120 bpm, blood pressure of 128/67 mm Hg, respiratory rate of 26/minute, oxygen saturation of 95% on room air, and temperature of 99.1°F. Physical examination reveals tachycardia and mild wheezing. An ECG is obtained and is shown above. D-dimer is 568 µg/mL. What is the best next step? A. Draw CBC, blood cultures, and order a chest X-ray B. Draw serum troponin C. Order CT pulmonary angiography D. Order venous ultrasound of lower extremities

C. Order CT pulmonary angiography

A 50-year-old man presents to the clinic with loose, greasy, and foul-smelling stools for the past 3 months. He reports chronic upper abdominal pain that radiates to his back. The pain resolves with acetaminophen or naproxen. He reports a medical history of hypertension, for which he takes amlodipine, and states he has been drinking a pint of vodka per day for the past 30 years. Vital signs include HR of 80 bpm, BP of 135/80 mm Hg, RR of 20/min, oxygen saturation of 98% on room air, and T of 98.6°F. Physical examination reveals a regular heart rate and rhythm without murmurs, rubs, or gallops. His skin is warm and dry without jaundice. He has minimal epigastric tenderness with no guarding or rebound tenderness. Laboratory studies are included below. ​​White blood cell count: 10,000 white blood cells per microliter Lipase: 90 U/L Total bilirubin: 1.0 mg/dL Aspartate aminotransferase: 30 U/L Alanine aminotransferase: 30 U/L Glucose: 130 mg/dL Immunoglobulin A anti-tissue transglutaminase: negative A recent CT scan of the abdomen and pelvis is shown above. Which of the following is the recommended treatment? A. Lactase enzyme supplementation B. Oral morphine C. Pancreatic enzyme replacem

C. Pancreatic enzyme replacement therapy

A 65-year-old woman with a medical history of hypertension and hyperlipidemia presents to the emergency department from home for headache and confusion. On presentation, she keeps grabbing her head, is unable to answer questions appropriately, and begins throwing up. She takes lisinopril 5 mg daily and atorvastatin 20 mg daily. Vital signs show a blood pressure of 200/100 mm Hg, heart rate of 100 bpm, temperature of 97°F, oxygen saturation of 98% room air, and respiratory rate of 18/minute. Neurological exam reveals the patient is oriented to person only. Her face is symmetrical and speech is normal, and she has 5/5 strength of bilateral upper and lower extremities. She is unable to complete most of the exam due to pain. A CT of the head shows no signs of infarction or bleed. A CT of the abdomen is negative for aneurysm. Which of the following is the appropriate treatment approach based on the most likely diagnosis? A. Do not reduce blood pressure unless it increases to 220/120 mm Hg B. Rapidly reduce systolic blood pressure to a target of 100-120 mm Hg C. Reduce blood pressure by no more than 20% in the first hour D. Reduce blood pressure by no more than 50% in the first hour

C. Reduce blood pressure by no more than 20% in the first hour

A 65-year-old man presents to the clinic for a follow-up of acute diverticulitis. During a recent hospitalization for acute diverticulitis, a CT of the abdomen and pelvis showed an abdominal aortic aneurysm of 5.7 cm. He is unsure whether there is any family history of an abdominal aortic aneurysm. He has a history of hypertension and type 2 diabetes mellitus. He is currently taking lisinopril 10 mg daily, metformin 1,000 mg twice daily, and glipizide 5 mg once daily. The patient appears well on exam. Vital signs show a blood pressure of 130/85 mm Hg, temperature of 97°F, heart rate of 85 bpm, and oxygen saturation of 98% room air. The cardiac exam reveals a regular rate and rhythm with no murmurs, 2+ distal pedal pulses bilaterally, and 2+ radial pulses bilaterally. The abdomen is soft and nondistended with mild tenderness to palpation in the left lower quadrant without guarding or rebound, and there are no palpable pulses or masses. What is the most appropriate next step based on the patient's diagnosis? A. Initiate medical treatment with a beta-blocker B. Initiate medical treatment with aspirin and a statin C. Request surgical consultation for elective repair D. Schedule surveillan

C. Request surgical consultation for elective repair

A 22-year-old man with a medical history of tobacco use presents to the emergency department for evaluation of acute onset shortness of breath. He states his symptoms started 2 hours prior to arrival. He has had no previous episodes and reports no chest pain, cough, fever, known sick contacts, nausea, or vomiting. He smokes five cigarettes per day and has for the last 4 years. Vital signs show a temperature of 98.4°F, heart rate of 120 bpm, blood pressure of 114/78 mm Hg, respiratory rate of 22 breaths/minute, and oxygen saturation of 90% on room air. His height is 6'2", weight is 170 lb, and BMI is 21.5 kg/m2. What is the most likely etiology for his condition? A. Alpha-1 antitrypsin deficiency B. Blunt thoracic trauma C. Rupture of subpleural apical bullae D. Underlying lung malignancy

C. Rupture of subpleural apical bullae

A 64-year-old man with a medical history of hypertension and diabetes mellitus is being admitted to the hospital after presenting with chest pain. The patient describes the pain as a substernal pressure that is provoked by walking more than a block. He has associated nausea and shortness of breath during the pain. Vital signs include a BP of 145/102 mm Hg, HR of 110 bpm, RR of 20/min, T of 98.6°F, and SpO2 of 99% on room air. On physical exam, the patient is clutching his chest and is diaphoretic. His ECG is shown above. Which of the following exam findings is common during the acute phase of the suspected condition? A. Early systolic ejection sound B. S3 sound C. S4 sound D. Wide splitting of S1

C. S4 sound

A 55-year-old man presents to the clinic with shortness of breath that has been present for the past 2 years. He feels tired throughout the day, and his spouse reports he snores loudly at night and intermittently gasps for air. He reports no chest pain, leg swelling, or wheezing. The patient has a medical history of diabetes mellitus, for which he takes metformin and glyburide. His BMI is 44 kg/m2, and he has never smoked cigarettes. Vital signs include HR of 95 bpm, BP of 140/90 mm Hg, RR of 22/min, oxygen saturation of 93% on room air, and T of 98.6°F. Physical examination reveals lungs clear to auscultation and no peripheral edema. The patient has a large neck with no jugular venous distention. He has a regular rate and rhythm with no murmurs. Chest X-ray shows elevation of both hemidiaphragms. Pulmonary function testing reveals a restrictive pattern. Transthoracic echocardiogram reveals normal ejection fractions, pulmonary arterial pressure of 41 mm Hg, and evidence of right ventricular enlargement. Which of the following laboratory findings would support the suspected diagnosis? A. B-type natriuretic peptide of 3,000 pg/mL B. Potassium of 3.1 mEq/L C. Serum bicarbonate of 32 mEq/

C. Serum bicarbonate of 32 mEq/L

A 62-year-old woman presents to the clinic with pain and redness on her right posterior calf for the past 2 days. She has a medical history of varicose veins and hypertension, for which she takes chlorthalidone. Vital signs include a BP of 140/85 mm Hg, HR of 80 bpm, RR of 20/min, T of 98.6°F, and SpO2 of 98% on room air. Physical examination findings include a regular heart rate and rhythm, localized erythema, and tenderness ascending vertically along the course of a varicose vein. There is no palpable cord, and the circumference of the lower extremities is symmetric. Which of the following is the suspected diagnosis? A. Cellulitis B. Popliteal cyst C. Superficial phlebitis D. Suppurative thrombophlebitis

C. Superficial phlebitis

A 62-year-old woman is admitted to the hospital after presenting to the emergency department with substernal chest pain. She reports she has had the chest pain intermittently for a week, but it has persisted today. She describes the pain as a chest-pressure sensation that radiates to her right arm and is worse with exertion. Her chest pain resolved after she was given aspirin, heparin, and nitroglycerin in the emergency department. She has a medical history of hyperlipidemia, for which she takes atorvastatin, and diabetes mellitus, for which she is on metformin and liraglutide. Vital signs include a BP of 130/88 mm Hg, HR of 70 bpm, RR of 20/min, T of 98.6°F, and SpO2 of 98% on room air. Physical examination reveals a regular rate and rhythm without murmurs, rubs, or gallops and clear lungs on bilateral auscultation. The patient has no peripheral edema. Her ECG is shown above. Her initial troponin I was 0.9 ng/mL, and the second troponin I from 2 hours later was 3.5 ng/mL. She undergoes coronary angiography, and her case is amenable to percutaneous coronary intervention. In addition to aspirin, which blood-thinning medication should this patient be on after percutaneous coronary interven

C. Ticagrelor

A 35-year-old woman with a medical history of chlamydia treated with doxycycline presents to the clinic with dysuria and urinary urgency for the past 6 days. Vital signs today include a heart rate of 86 bpm, blood pressure of 118/76 mm Hg, respiratory rate of 20/minute, oxygen saturation of 98% on room air, and temperature of 99°F. Physical exam reveals suprapubic tenderness and no flank pain. Dipstick urinalysis is positive for nitrite. Which of the following carries the highest risk for the suspected condition? A. History of bacterial vaginosis B. History of sexual abstinence C. Use of diaphragms D. Use of non-spermicide-coated condoms

C. Use of diaphragms

A 24-year-old man presents to the clinic reporting coughing, feeling short of breath, and wheezing that starts about 5 minutes after exercising. He recently developed these symptoms after joining his college ice hockey team and mentions his symptoms worsen as his training intensifies but will resolve after rest. He has no significant medical history of any illnesses and states he never experienced these symptoms when he played basketball recreationally with his friends. He does not normally experience cough or wheezing when he goes about his daily tasks. He is not currently taking any medications. Vital signs include a heart rate of 60 bpm, blood pressure of 120/80 mm Hg, temperature of 98.6°F, RR of 18/min, and oxygen saturation of 99% on room air. His physical exam findings reveal a normal cardiac exam without murmurs and lungs that are clear to auscultation. Spirometry is performed at rest and 10 minutes post treadmill exercise. Spirometry performed post exercise has a FEV1 that is 10% lower compared to the FEV1 performed at rest. How should this patient's condition be managed? A. Use of inhaled corticosteroids daily B. Use of inhaled corticosteroids-long-acting beta-agonist daily C.

C. Use of short-acting beta-agonist 5-20 minutes before exercise

A 24-year-old woman presents to the gastroenterologist for evaluation of 3 months of diarrhea, flatulence, weight loss, and pruritic rash. She also reports intermittent abdominal discomfort and bloating. The patient reports no dietary changes, recent travel, or new medications. Vital signs include a temperature of 98.5°F, blood pressure of 122/76 mm Hg, heart rate of 78 bpm, respiratory rate of 16/minute, and oxygen saturation of 98% on room air. Physical examination reveals regular heart rate and rhythm, clear lungs bilaterally on auscultation, soft and nontender abdomen on palpation, and a papulovesicular rash on the elbows, knees, and scalp with excoriations. Which laboratory finding is most consistent with the suspected diagnosis? A. Anti-centromere antibody B. Anti-mitochondrial antibody C. Anti-smooth muscle antibody D. Anti-tissue transglutaminase antibody

D. Anti-tissue transglutaminase antibody

An 18-year-old woman presents to the clinic with increased white and yellow vaginal discharge for 3 days. She has normal menstrual periods. The patient reports that she has had two new sexual partners in the past month without protection. Vital signs include HR of 80 bpm, BP of 120/80 mm Hg, RR of 20/min, oxygen saturation of 99% on room air, and T of 98.6°F. Physical examination reveals a regular heart rate and rhythm and lungs clear to auscultation bilaterally. Her abdomen is soft and nontender. She has white vaginal discharge on exam, and her cervix is friable, but she has no cervical motion tenderness. Wet prep reveals no clue cells or motile trichomonads. Pregnancy test is negative. Which of the following is the recommended empiric treatment? A. Azithromycin 1 g orally for one dose B. Azithromycin 1 g orally for one dose and metronidazole 500 mg twice daily for 7 days C. Ceftriaxone 500 mg IM for one dose and azithromycin 1 g orally for one dose D. Ceftriaxone 500 mg IM for one dose and doxycycline 100 mg twice daily for 7 days

D. Ceftriaxone 500 mg IM for one dose and doxycycline 100 mg twice daily for 7 days

A 58-year-old man presents to the clinic with fatigue, weight loss, night sweats, and abdominal fullness. Physical exam reveals splenomegaly, sternal tenderness, and multiple bruises on the upper extremities. Laboratory testing reveals a white blood cell count of 105,000 cells/microL with increased counts of all neutrophilic cells, 1% blasts, as well as thrombocytosis and normocytic anemia. Which of the following is the most likely diagnosis? A. Acute lymphoblastic leukemia B. Acute myeloid leukemia C. Chronic lymphocytic leukemia D. Chronic myelogenous leukemia

D. Chronic myelogenous leukemia

A 67-year-old man is evaluated in the emergency department for difficulty walking. Symptoms began approximately 1 day ago. The patient notes that his left leg feels weak. He is also experiencing incontinence. His medical history includes chronic obstructive pulmonary disease, for which he uses a tiotropium inhaler. His vital signs reveal a BP of 168/102 mm Hg, HR of 92 bpm, RR of 18/min, T of 98.4°F, and SpO2 of 95%. Physical examination demonstrates decreased strength and sensation in the left leg. Deep tendon reflex testing reveals upper extremities and right lower extremity as 2+ and left lower extremity as 4+. Gait apraxia is demonstrated. A CT scan of the head is performed and included above. What risk factor is most strongly associated with the suspected diagnosis? A. Diabetes mellitus B. History of smoking C. Hyperlipidemia D. Hypertension

D. Hypertension

A 24-year-old woman presents to the clinic with dyspnea, phlegm production, cough, and wheezing. She reports some dusty occupational exposure but denies any cigarette smoking. Her father developed chronic obstructive pulmonary disease at an early age despite no history of cigarette smoking. These observations suggest that the disease is a result of which of the following mechanisms? A. Abnormal dilation and bronchial wall destruction B. Enlargement of air spaces and alveolar wall destruction C. Excessive secretion of bronchial mucus D. Imbalance between lung neutrophil elastase and alpha-1 antitrypsin

D. Imbalance between lung neutrophil elastase and alpha-1 antitrypsin

A 53-year-old woman presents to the clinic with new dysphagia, odynophagia, and retrosternal pain for 2 days. Her medical history includes well-controlled type 2 diabetes mellitus, hypertension, left atrial enlargement, osteoarthritis, and gastroesophageal reflux disease, which are treated with famotidine, lisinopril, metformin, and naproxen. Vital signs include a heart rate of 76 bpm, blood pressure of 128/78 mm Hg, respiratory rate of 20/minute, oxygen saturation of 99% on room air, and temperature of 98.6°F. Electrocardiography indicates normal sinus rhythm. Physical examination reveals a regular rate and rhythm, lungs clear to auscultation, and no chest wall or abdominal tenderness to palpation. Which of the following medications is most likely the cause of her symptoms? A. Famotidine B. Lisinopril C. Metformin D. Naproxen

D. Naproxen

A 62-year-old woman with a medical history of obesity, hyperlipidemia, and hypertension presents to the clinic for her annual checkup. Patient states she takes multiple medications and was recently started on a new medication for hyperlipidemia, but she cannot recall any of her medication names. She reports that sometimes she feels her face flush, and her family members have pointed it out as well. She says this is new for her and started about 1 month ago. Vital signs include a heart rate of 89 bpm, blood pressure of 134/81 mm Hg, respiratory rate of 16/minute, oxygen saturation of 99% on room air, and temperature of 98.4°F. Upon physical examination, there are no lifts, heaves, or thrills identified on palpation of the chest. The S1 and S2 heart sounds are in regular rhythm with no murmurs noted. Her skin appears warm, pink, and moist. Her capillary refill is < 2 seconds in the upper and lower extremities. Her recent fasting labs show a blood glucose of 167 mg/dL, hemoglobin A1C of 5.9%, LDL of 152 mg/dL, and an HDL of 34 mg/dL. Which of the following medications is most likely to be causing her symptoms? A. Atorvastatin B. Ezetimibe C. Gemfibrozil D. Niacin

D. Niacin

A man presents to the clinic complaining of fever, cough, and shortness of breath. He states his symptoms began after his return from a business trip last week. He has also experienced nausea, vomiting, and diarrhea since returning. Chest radiograph demonstrates patchy unilobar infiltrates. Which of the following results do you expect to see on Gram stain? A. Gram-negative encapsulated bacilli B. Gram-positive cocci in clusters C. Gram-positive diplococci D. No bacteria present

D. No bacteria present

A 40-year-old woman presents to the clinic with abdominal cramping, diarrhea, and fever for 2 days. The symptoms started a day after she consumed chicken that may not have been cooked completely. The patient reports no vomiting. She has a medical history of hypertension, for which she takes lisinopril. Vital signs include HR of 96 bpm, BP of 120/80 mm Hg, RR of 20/min, oxygen saturation of 98% on room air, and T of 99.9°F. Physical examination reveals a nontoxic-appearing woman with mild tachycardia and lungs that are clear to auscultation. The patient's abdomen is soft and nontender with hyperactive bowel sounds. Her mucous membranes are slightly dry, and her skin turgor is brisk. An enteric pathogens polymerase chain reaction panel is positive for Salmonella. What is the recommended treatment? A. Intravenous fluids B. Oral azithromycin C. Oral ciprofloxacin D. Oral rehydration solution

D. Oral rehydration solution

A 45-year-old man with a history of intravenous drug use is admitted to the hospital after presenting with an insidious onset of dyspnea, cough, and night sweats over the past 2 weeks. The patient also has had increased fatigue and lower extremity edema during the past couple of days prior to hospitalization. His initial chest X-ray showed bilateral upper lobe infiltrates with a cavitary lesion and enlarged cardiac silhouette. Vital signs include a BP of 90/70 mm Hg, HR of 175 bpm, RR of 28/min, T of 100.6°F, and SpO2 of 94% on 2 L nasal cannula. The patient's ECG is shown above. Echocardiogram shows chamber collapse and exaggerated respiratory variation of cardiac and venous flow. Which of the following is the recommended treatment? A. Intravenous thrombolytics B. Intubation for mechanical ventilation and IV furosemide C. Nonsteroidal anti-inflammatory drugs D. Pericardiocentesis

D. Pericardiocentesis

A 62-year-old woman is admitted to the hospital after a syncopal episode earlier today. She reports generally feeling fatigued and dyspneic and is lightheaded upon standing. She has a history of hyperlipidemia, for which she takes atorvastatin. Vital signs include a BP of 80/50 mm Hg, RR of 22/min, T of 98.6°F, and SpO2 of 99% on room air. Physical examination reveals a woman who is diaphoretic and pale with irregular pulses and lungs that are clear to auscultation. She has no peripheral edema on examination. Her ECG is shown above. She is stabilized on the next recommended treatment, and you cannot find a reversible cause of her dysrhythmia. Which of the following is the recommended next step in treatment? A. Atropine administration B. Implantable cardioverter-defibrillator placement C. No additional treatment D. Permanent pacemaker placement

D. Permanent pacemaker placement

A 64-year-old woman with a history of hypertension and type 2 diabetes mellitus presents to the emergency department with 2 days of vomiting and diarrhea. She reports recently returning from a cruise and thinks she ate contaminated food. Vital signs today include a heart rate of 98 bpm, blood pressure of 102/59 mm Hg, respiratory rate of 18/minute, oxygen saturation of 98% on room air, and temperature of 98.4°F. Physical examination reveals a regular rate and rhythm, no stridor or signs of respiratory distress, and mild diffuse abdominal tenderness to palpation. Lab work reveals a blood urea nitrogen/creatinine ratio of > 20:1 and a fractional excretion of sodium < 1%. Which of the following best characterizes this patient's presentation? A. Chronic kidney disease B. Intrinsic kidney failure C. Postrenal azotemia D. Prerenal azotemia

D. Prerenal azotemia

A 35-year-old man presents to the office for a routine physical examination. He is otherwise healthy and does not take any medications. However, he reports increasing headaches and numbness and tingling in his hands and feet that have gotten progressively worse over the past few months. His temperature is 98.6°F, blood pressure is 176/102 mm Hg, heart rate is 86 bpm, oxygen saturation is 98% on room air, and respiratory rate is 21/minute. His physical exam is unremarkable. His laboratory results are as follows: a sodium level of 146 mEq/L, a potassium level of 2.9 mEq/L, and a sodium bicarbonate level of 37 mEq/L. Which of the following is the most likely diagnosis? A. Addison disease B. Cushing syndrome C. Pheochromocytoma D. Primary aldosteronism

D. Primary aldosteronism

A 45-year-old woman presents to the clinic with dysuria that has lasted 3 months. She reports she has been seen in several urgent care centers and treated with antibiotics multiple times without relief. She has a medical history of irritable bowel syndrome, for which she is treated with dicyclomine. Vital signs include a BP of 120/82 mm Hg, HR of 80 bpm, RR of 20/minute, T of 98.6°F, and SpO2 of 98% on room air. Physical examination findings include a regular rate and rhythm with lungs clear to auscultation. On exam, the patient has suprapubic and pelvic floor tenderness with no pelvic organ prolapse. Urinalysis is negative for pyuria, leukocyte esterase, and nitrites. Urine culture is negative. The patient's postvoid residual urine is 40 mL. Cystoscopy is performed and shows reddened lesions on the bladder mucosa with attached fibrin deposits. Which of the following is the most characteristic clinical manifestation of the suspected condition? A. Bladder pain with hematuria B. Frequent voiding to avoid incontinence C. Postvoid dribbling of urine D. Relief of bladder discomfort with voiding

D. Relief of bladder discomfort with voiding

A 74-year-old woman has been in the hospital for an upper respiratory infection when she reports increasing left knee pain and swelling without inciting injury. She reports no history of prior knee problems. She has no significant medical history. She follows a vegan diet and exercises daily with running and yoga. She reports no history of drinking alcohol or smoking cigarettes. She is afebrile, and vital signs are unremarkable. Physical examination of the left knee reveals a large effusion, no erythema, and no warmth. She ambulates with a mildly antalgic gait and is able to perform left knee range of motion 5-95° with moderate but tolerable pain. X-ray reveals no acute fracture, dislocation, or significant arthritis, but the joint effusion is appreciated as well as some calcified-appearing radiodensities within the joint space. Aspiration of the knee reveals low-viscosity synovial fluid with a pale yellow, opaque appearance. Laboratory analysis of the synovial fluid is pending. Which of the following pertinent laboratory findings is most likely to be found in this patient? A. Decreased synovial fluid glucose levels B. Elevated synovial fluid leukocyte count C. Needle-shaped crystals th

D. Rhomboid-shaped crystals that are positively birefringent

A 78-year-old man presents with fatigue that has been ongoing for the past few days. He reports lightheadedness with exertion and exercise intolerance. He reports several presyncopal episodes that occur with exertion. Vital signs are a heart rate of 49 bpm, blood pressure of 113/67 mm Hg, oxygen saturation of 99%, respiratory rate of 16/min, and temperature of 98.2°F. Auscultation of heart sounds reveals an irregular rhythm. An electrocardiogram is shown above. Which of the following is the most likely cause of the diagnosis? A. Hypothyroidism B. Rheumatic fever C. Sinoatrial nodal artery disease D. Sinus node fibrosis

D. Sinus node fibrosis

A 55-year-old man presents with nausea, vomiting, lethargy, weakness, and reduced urinary output for the past few days. A review of systems reveals that he has been experiencing left-sided chest pain that has been a constant ache for the past month. His medical history is significant for hyperlipidemia Social history is significant for tobacco use of 1 pack of cigarettes per day for 30 years. Vital signs are heart rate of 76 bpm, blood pressure of 132/76 mm Hg, temperature of 98.9°F, and oxygen saturation of 92%. Lab tests are completed and reveal a serum sodium of 122 mEq/L and a urine sodium of 56 mEq/L. A chest radiograph is completed and demonstrates a large, left hilar mass. Which of the following is the most likely diagnosis? A. AdenocarcinomaYour Answer B. Large cell lung cancer C. Pancoast tumor D. Small cell lung cancer

D. Small cell lung cancer

A 38-year-old man with a medical history of childhood asthma and recurrent tonsillopharyngitis presents to the clinic with 3 days of exertional dyspnea. He does not take any medications and does not have access to a primary care clinician in his hometown. Vital signs today include a heart rate of 76 bpm, blood pressure of 110/78 mm Hg, respiratory rate of 22/minute, oxygen saturation of 96% on room air, and temperature of 98.4°F. Physical exam reveals a quick, loud, second heart sound followed by a low rumbling diastolic murmur. Chest radiograph is normal. Which of the following is the most likely cause of this patient's symptoms? A. Coxsackievirus infection B. Epstein-Barr virus infection C. Lyme disease D. Streptococcal infection

D. Streptococcal infection

A 72-year-old woman with a history of depression and hyperlipidemia presents to the clinic with a productive cough for 1 week. She takes escitalopram 10 mg daily and atorvastatin 20 mg daily. The patient reports pain in her chest when she coughs and when she inhales. She also reports increasing shortness of breath. Her vital signs are a heart rate of 107 bpm, blood pressure of 132/83 mm Hg, respiratory rate of 16/minute, oxygen saturation of 95% on room air, and temperature of 102.4°F. Upon physical exam, an inspection of the chest is unremarkable, late inspiratory crackles are heard at the base of the right lower lobe, and there is increased tactile fremitus. The S1 and S2 heart sounds are in regular rhythm and no murmurs are heard. What is the most common etiology of this patient's condition? A. Klebsiella pneumoniae B. Legionella pneumophila C. Mycoplasma pneumoniae D. Streptococcus pneumoniae

D. Streptococcus pneumoniae

A 65-year-old man presents with a productive cough and dyspnea on exertion that has been worsening over the past few months. When questioned, he reports that he has had some mild dyspnea on exertion and a cough due to smoking for several years. He reports tobacco use and has smoked one pack per day for the last 40 years. He reports no recent illnesses, fever, hemoptysis, or chest pain. He reports no weight loss or changes in appetite. His vital signs are a heart rate of 86 bpm, blood pressure of 138/82 mm Hg, temperature of 97.9°F, and oxygen saturation of 91%. Spirometry is performed and reveals an obstructive pattern with a decreased FEV1, an increased residual volume, and an increased total lung capacity. Considering the most likely diagnosis, which of the following treatment options has been associated with decreased mortality? A. Inhaled fluticasone B. Inhaled ipratropium bromide C. Oral theophylline D. Supplemental oxygen therapy

D. Supplemental oxygen therapy

A 72-year-old man presents to the clinic reporting fatigue, dizziness, confusion, and shortness of breath at rest for 2 days. His medical history is significant for hypertension managed on lisinopril for 6 years and type 2 diabetes mellitus managed on metformin for 4 years. Vital signs are T 99.2°F, BP 90/50 mm Hg, HR 44 bpm, RR 16 breaths/min, and SpO2 96% on room air. An ECG study is shown above. Blood glucose is 90 mg/dL. A total of 3 mg of atropine has been administered with no improvement. What is the most appropriate intervention? A. Administer IV glucagon B. Administer IV norepinephrine C. Electrical cardioversion D. Temporary cardiac pacing

D. Temporary cardiac pacing

A 40-year-old woman presents to the clinic for further evaluation of a thyroid nodule that was identified incidentally when the patient had a chest CT performed in the emergency department. The patient reports no voice changes, dysphagia, or dyspnea. Vital signs include a BP of 120/80 mm Hg, HR of 80 bpm, RR of 20/minute, T of 98.6°F, and SpO2 of 99% on room air. Physical examination reveals a nodule on the right side of the thyroid. The patient's thyroid-stimulating hormone is 3.0 mIU/L. What is the next recommended step for this patient? A. CT scan of the neck B. Fine-needle aspiration C. Surgical lobectomy D. Thyroid ultrasonography

D. Thyroid ultrasonography

A 25-year-old woman with a medical history of schizophrenia treated with olanzapine presents to the clinic with a rash that has progressively worsened over the last 3 weeks. Vital signs today include a heart rate of 98 bpm, blood pressure of 130/92 mm Hg, respiratory rate of 20/minute, oxygen saturation of 98% on room air, and temperature of 98.2°F. Physical exam reveals discrete, erythematous, somewhat indurated plaques that are photodistributed on the dorsal arms and face. Follicular plugging is noted on the scalp, and mild bilateral effusions are noted on the metacarpal, olecranon, and patellar joints. Routine dipstick urinalysis reveals 3+ proteinuria. In addition to recommending sun protective measures, administration of which of the following medications is the most appropriate next step to treat this patient's skin lesions? A. Hydroxychloroquine B. Intralesional corticosteroids C. Topical calcineurin inhibitors D. Topical clobetasol

D. Topical clobetasol

A 78-year-old man is admitted to the hospital for an exacerbation of chronic obstructive pulmonary disease after presenting with a cough and shortness of breath. He was noted to be tachycardic at the time of admission. He states his shortness of breath seems to be improving some after corticosteroids and bronchodilators in the emergency department. He reports no palpitations, chest pain, or lightheadedness. The patient has a history of diabetes mellitus, for which he takes metformin and glyburide, and chronic obstructive pulmonary disease, for which he takes inhaled corticosteroids and bronchodilators. His latest vital signs include a BP of 142/90 mm Hg, HR of 125 bpm, RR of 24/min, T of 99.4°F, and SpO2 of 93% on 2 liters of oxygen by nasal cannula. Physical exam findings include an irregular tachycardic heart rate, scattered expiratory wheezing on lung auscultation, and no peripheral edema of either lower extremity. A CT pulmonary angiogram is performed and is negative for pulmonary embolism or pneumonia. His ECG is shown above. What is the recommended treatment for the cause of his tachycardia? A. Administering intravenous diltiazem B. Initiating carotid sinus massage C. Performing e

D. Treating the underlying condition

A 58-year-old woman presents to the clinic due to fatigue for 1 month and one episode of syncope after jogging. Her medical history is significant for rheumatic heart disease. Vital signs are T 98.6°F, BP 118/80 mm Hg, HR 80 bpm, RR 14 breaths/min, and SpO2 99% on room air. The physical exam reveals lower extremity edema, jugular venous distension, hepatomegaly, and a low-frequency mid-diastolic murmur heard best at the lower left sternal border that increases with inspiration. Which of the following is the most likely diagnosis? A. Aortic regurgitation B. Mitral regurgitation C. Tricuspid regurgitation D. Tricuspid stenosis

D. Tricuspid stenosis

In a patient with subarachnoid hemorrhage, what develops as a result of lysis of the subarachnoid blood clots and release of nitrous oxide and endothelin approximately three days after onset of symptoms? A. Increased intracranial pressure B. Meningismus C. Seizure D. Vasospasm

D. Vasospasm

A 52-year-old man presents to the clinic with chest pain that started this morning. He describes the pain as sharp, worse with inspiration, and improved when leaning forward. He reports no shortness of breath or leg swelling. He has a history of depression, for which he takes fluoxetine. Vital signs include a BP of 120/80 mm Hg, HR of 100 bpm, RR of 20/min, T of 99.6°F, and SpO2 of 98% on room air. Physical examination reveals clear lungs on bilateral auscultation, regular heart rate and rhythm, and a superficial scratchy heart sound heard best over the left sternal border. He has no peripheral edema and 2+ distal pulses. His ECG is shown above. His troponin I is 0.2 ng/mL. Which of the following is the most likely cause of this patient's suspected condition? AAutoimmune disease BMyocardial infarction CNeoplastic disease DViral infection

D. Viral infection


संबंधित स्टडी सेट्स

English Midterm 11 Study Guide -Unit 4

View Set

Math - Introduction to Functions

View Set

Global Sports and National Culture Exam 1

View Set

Programming Fundalmentals II — Test 4

View Set

ACCT 101 Ch. 12 THE STATEMENT OF CASH FLOWS 2017

View Set

Life insurance basic Chapter quiz

View Set

***HURST Review Elevate Q-Cards***

View Set

Module 9 - Conductors and Cables

View Set